Đến nội dung

Hình ảnh

Cauchy-Schwarz


  • Please log in to reply
Chủ đề này có 176 trả lời

#41
pacific_boy571

pacific_boy571

    Lính mới

  • Thành viên
  • 9 Bài viết
Có một bài này mà em chưa nghĩ ra. Bác nào nghĩ hộ em cái.
Cho cho a,b,c>0 và thỏa mãn $a^{2}+ b^{2}+c^{2}=1$.
Tìm giá trị bé nhất của:
$P= \dfrac{a}{b^{2}+c^{2}} + \dfrac{b}{c^{2}+a^{2}} + \dfrac{c}{a^{2}+b^{2}}$

Bài viết đã được chỉnh sửa nội dung bởi Tham Lang: 28-07-2012 - 17:39

<span style='font-family:Times'><span style='font-size:14pt;line-height:100%'><span style='color:purple'><center>LAM SƠN - TÌNH YÊU VÀ NỖI NHỚ - CHẮP CÁNH ƯỚC MƠ.</center></span></span></span>

#42
NguLauDotBen

NguLauDotBen

    Binh nhì

  • Thành viên
  • 15 Bài viết

- Nó cũng có một số hệ quả:
1, Bất đẳng thức Schwarz:
Với hai dãy số thực $(a_{1}, a_{2}, ..., a_{m})$ và $(b_{1}, b_{2}, ..., b_{m})$ sao cho $b_{i} \geq 0$ ta luôn có bất đẳng thức:
$\dfrac{a_{1}^2}{b_{1}}+ \dfrac{a_{2}^2}{b_{2}}+...+ \dfrac{a_{m}^2}{b_{m}} \geq \dfrac{(a_{1}+a_{2}+...+a_{m})^2}{b_{1}+b_{2}+...+b_{m}}$

Ủa anh ơi làm sao chứng minh dc hệ quả này zậy anh

Bài viết đã được chỉnh sửa nội dung bởi bboy114crew: 19-08-2011 - 11:28


#43
Toanlc_gift

Toanlc_gift

    Sĩ quan

  • Hiệp sỹ
  • 315 Bài viết

Ủa anh ơi làm sao chứng minh dc hệ quả này zậy anh

chứng minh như vầy em ạ:
áp dụng với 2 bộ số:
Bộ thứ nhất:
$\dfrac{{{a_1}}}{{\sqrt {{b_1}} }};\dfrac{{{a_2}}}{{\sqrt {{b_2}} }};...\dfrac{{{a_n}}}{{\sqrt {{b_m}} }}$
Bộ thứ hai:
$\sqrt {{b_1}} ;\sqrt {{b_2}} ...\sqrt {{b_m}} $
việc còn lại là nhân tích chéo thôi :D

=.=


#44
NguLauDotBen

NguLauDotBen

    Binh nhì

  • Thành viên
  • 15 Bài viết
Dạ em đã cm dc rồi ths anh nhiều

Bài viết đã được chỉnh sửa nội dung bởi NguLauDotBen: 03-06-2009 - 22:03


#45
Messi_ndt

Messi_ndt

    Admin batdangthuc.com

  • Thành viên
  • 679 Bài viết

Dạ em đã cm dc rồi ths anh nhiều

Bài này ai dùng CBS chém thử nè:
Cho $ a,b,c \geq 0;a^2+b^2+c^2=3 $.Chứng minh rằng:$ a^2b^3+b^2c^3+c^2a^3 \leq 3 $.
Không dùng langrage;hoán vị. :D :D

#46
toanhoc10

toanhoc10

    Binh nhì

  • Thành viên
  • 13 Bài viết

ấn tượng mãi cái bài của bác LEE hojoo
(a,b,c>0)cm
$\sum\limits_{cyc} \sqrt{a^4+a^2b^2+b^4} \geq \sum\limits_{cyc} a\sqrt{2a^2+bc}$

ko bik học phổ thông các thầy có cho ứng dụng thẳng cái này ko cà

Bài viết đã được chỉnh sửa nội dung bởi bboy114crew: 19-08-2011 - 11:22


#47
toanhoc10

toanhoc10

    Binh nhì

  • Thành viên
  • 13 Bài viết

Thử bài này xem cho a,b,c dương thỏa ab+bc+ca=1 CMR:
$\large\dfrac{1}{a+b}+\dfrac{1}{b+c}+\dfrac{1}{c+a} \geq \dfrac{5}{2} $ (D�ồn biến ko dành cho THCS)

anh cho em xin tất cả các BDT có thể thi Dh dể em swallow dc hem?

Bài viết đã được chỉnh sửa nội dung bởi bboy114crew: 19-08-2011 - 11:22


#48
beatboxbup

beatboxbup

    Lính mới

  • Thành viên
  • 4 Bài viết

Bài 1 hơi dễ chút xíu nhưng sử dụng Cauchy thì sao nhỉ?
Nếu x, y trái dấu. Suy ra đpcm.
Nếu x, y cùng dấu, biến đổi BDT thành $ (x - y)^2(xy+1) \geq 0$
Bài 2 cũng dễ dễ làm luôn vậy :)
Đưa về CM:
$ 2\sqrt{2}(\sqrt{x+9}+\sqrt{x}) \leq 9\sqrt{x+1}$

$ \Leftrightarrow 8(\sqrt{x+9}+\sqrt{x})^2 \leq 8(1+\dfrac{1}{8})(x+9+8x)=81(x+1)$

Ta có điều phải chứng minh.

Hoặc có thể chứng minh như sau.
Theo đề bài, 2 số đã cho luôn dương.
Áp dụng BĐT Côsi cho 2 số không âm
Ta được BĐT tương đương $ (x - y)^2 \geq 0$
Điều này luôn đúng nên ta có điều phải chứng minh.

Bài viết đã được chỉnh sửa nội dung bởi Phạm Hữu Bảo Chung: 18-08-2011 - 21:48


#49
maikhai

maikhai

    Hạ sĩ

  • Thành viên
  • 52 Bài viết
Anh chứng minh cho em BĐT Schwarz đi, cùng BĐT Netbit nữa.

Mod: Gõ Tiếng Việt cho đúng nhé bạn.

Bài viết đã được chỉnh sửa nội dung bởi Phạm Hữu Bảo Chung: 18-08-2011 - 21:43

Đừng cười khi người khác bị vấp ngã!

Vì bạn cũng có thể vấp ngã giống như họ!



Ai ơi chớ vội cười người


Cười người hôm trước hôm sau người cười


#50
NguyThang khtn

NguyThang khtn

    Thượng úy

  • Hiệp sỹ
  • 1468 Bài viết

ấn tượng mãi cái bài của bác LEE hojoo
(a,b,c>0)cm
$\sum\limits_{cyc} \sqrt{a^4+a^2b^2+b^4} \geq \sum\limits_{cyc} a\sqrt{2a^2+bc}$

ko bik học phổ thông các thầy có cho ứng dụng thẳng cái này ko cà!
Bài này thực ra sử dụng một cái BDT quen thuộc!
$\sqrt{x^2+x2y+y^2} \geq \dfrac{\sqrt{3}(x+y)}{2}$
Với $x=a^2;y=b^2$
TA có:
$(\sum\limits_{cyc} \sqrt{a^4+a^2b^2+b^4})^2 \geq 3(\sum\limits_{cyc} a^2)^2$
Ap dụng BDT Cauchy-schwarz:
$(\sum\limits_{cyc} a\sqrt{2a^2+bc})^2 \leq (\sum\limits_{cyc}a^2)[\sum\limits_{cyc}(2a^2+bc)] \leq 3(\sum\limits_{cyc} a^2)^2$

Có một bài này mà em chưa nghĩ ra. Bác nào nghĩ hộ em cái.
Cho cho a,b,c>0 và thỏa mãn $a^{2}+ b^{2}+c^{2}=1$.
Tìm giá trị bé nhất của:
$P= \dfrac{a}{b^{2}+c^{2}} + \dfrac{b}{c^{2}+a^{2}} + \dfrac{c}{a^{2}+b^{2}}$

Típ!
Ta có:
$P= \dfrac{a}{b^{2}+c^{2}} + \dfrac{b}{c^{2}+a^{2}} + \dfrac{c}{a^{2}+b^{2}}= \sum\limits_{cyc} \dfrac{a}{1-a^2} $
Ap dụng BDT AM-GM:
$\Large 2x^2(1-x^2)(1-x^2) \le \dfrac{8}{27} \Rightarrow \dfrac{x}{1-x^2} \ge \dfrac{3\sqrt{3}}{2}x^2$ Tương tự . ta sẽ tìm được MIN!

Bài viết đã được chỉnh sửa nội dung bởi Tham Lang: 28-07-2012 - 17:40

It is difficult to say what is impossible, for the dream of yesterday is the hope of today and the reality of tomorrow

 


#51
Yagami Raito

Yagami Raito

    Master Tetsuya

  • Thành viên
  • 1333 Bài viết
Cho 3 số dương $ a,b,c $ thỏa mãn $ a+b+c=3 $. Chứng minh rằng

$\dfrac{a}{1+b^{2}}+\dfrac{b}{1+c^{2}}+\dfrac{c}{1+a^{2}} \geq 1.5$


Bài viết đã được chỉnh sửa nội dung bởi Phạm Hữu Bảo Chung: 19-08-2011 - 21:39

:nav: Học gõ công thức toán học tại đây

:nav: Hướng dẫn đặt tiêu đề tại đây

:nav: Hướng dẫn Vẽ hình trên diễn đàn toán tại đây

--------------------------------------------------------------

 


#52
phuonganh_lms

phuonganh_lms

    Thượng sĩ

  • Thành viên
  • 293 Bài viết
Cho 3 số dương $ a,b,c $ thỏa mãn $ a+b+c=3 $. Chứng minh rằng

$\dfrac{a}{1+b^{2}}+\dfrac{b}{1+c^{2}}+\dfrac{c}{1+a^{2}} \geq 1.5$


Ta có $ \dfrac{a}{1+b^2}=a-\dfrac{ab^2}{1+b^2} \ge a-\dfrac{ab^2}{2b}=a-\dfrac{ab}{2}$

Khi đó

$ \dfrac{a}{1+b^{2}}+\dfrac{b}{1+c^{2}}+\dfrac{c}{1+a^{2}}\ge (a+b+c)-\dfrac{1}{2}(ab+bc+ca) \ge(a+b+c)-\dfrac{1}{6}(a+b+c)^2=\dfrac{3}{2}$

Dấu = xảy ra khi $a=b=c=1$





Hình đã gửi


#53
phuonganh_lms

phuonganh_lms

    Thượng sĩ

  • Thành viên
  • 293 Bài viết
Bài trên dùng kĩ thuật cauchy ngược dấu. Đây là 1 kĩ thuật khá hiệu quả với các bdt hoán vị.
Để rèn luyện kĩ thuật trên, các bạn có thể giải các ví dụ đơn giản sau:

1. Mở rộng bài trên với 4 biến: $a,b,c,d>0$, $ a+b+c+d=4$
CMR:

$ \dfrac{a}{1+b^2}+\dfrac{b}{1+c^2}+\dfrac{c}{1+d^2}+\dfrac{d}{1+a^2} \ge 2$

2. ( Đề thi thử môn Toán lần 2- THPT Chuyên Nguyễn Huệ 2007-2008)
$a,b,c,d>0$, $ a+b+c=3$
CMR:

$\dfrac{a+b+c}{a^2+abc}+\dfrac{a+b+c}{b^2+abc}+\dfrac{a+b+c}{c^2+abc} \ge \dfrac{9}{2} $

3.$ a,b,c>0$, $ a+b+c=3$
CMR:

$\dfrac{a+1}{b^2+1}+\dfrac{b+1}{c^2+1}+\dfrac{c+1}{a^2+1} \ge 3$


Tương tự có thể cm bdt 3 với 4 biến

Bài viết đã được chỉnh sửa nội dung bởi phuonganh_lms: 20-08-2011 - 22:56

Hình đã gửi


#54
PRONOOBCHICKENHANDSOME

PRONOOBCHICKENHANDSOME

    Thượng sĩ

  • Thành viên
  • 227 Bài viết

Bài trên dùng kĩ thuật cauchy ngược dấu. Đây là 1 kĩ thuật khá hiệu quả với các bdt hoán vị.
Để rèn luyện kĩ thuật trên, các bạn có thể giải các ví dụ đơn giản sau:

1. Mở rộng bài trên với 4 biến: $a,b,c,d>0$, $ a+b+c+d=4$
CMR:

$ \dfrac{a}{1+b^2}+\dfrac{b}{1+c^2}+\dfrac{c}{1+d^2}+\dfrac{d}{1+a^2} \ge 2$

2. ( Đề thi thử môn Toán lần 2- THPT Chuyên Nguyễn Huệ 2007-2008)
$a,b,c,d>0$, $ a+b+c=3$
CMR:

$\dfrac{a+b+c}{a^2+abc}+\dfrac{a+b+c}{b^2+abc}+\dfrac{a+b+c}{c^2+abc} \ge \dfrac{9}{2} $

3.$ a,b,c>0$, $ a+b+c=3$
CMR:

$\dfrac{a+1}{b^2+1}+\dfrac{b+1}{c^2+1}+\dfrac{c+1}{a^2+1} \ge 3$


Tương tự có thể cm bdt 3 với 4 biến

1.CM tương tự : $VT\geq (a+b+c+d)- \dfrac{ab+bc+cd+da}{2}=4-\dfrac{(b+d)(c+a)}{2}\geq 4-\dfrac{\dfrac{(b+d+c+a)^2}{4}}{2}=4-2=2.$
3.Có $VT = \sum \dfrac{a}{b^2+1} +\sum \dfrac{1}{a^2+1} \geq \dfrac{3}{2}+\sum \dfrac{1}{a^2+1}.$
Mà $\dfrac{1}{a^2+1}=1-\dfrac{a^2}{a^2+1}\geq 1-\dfrac{a^2}{2a}=1-\dfrac{a}{2}$
$\therefore \sum \dfrac{1}{a^2+1}\geq 3-\dfrac{\sum a}{2}=\dfrac{3}{2}$
$\therefore VT\geq 3\therefore Q.E.D$

2. Có $a+b+c=3$
$\therefore abc\leq \dfrac{(a+b+c)^3}{27}=1$
$\therefore Q.E.D\Leftrightarrow \sum \dfrac{1}{a^2+1}\geq \dfrac{3}{2}$
Đã Cm ở trên

#55
Beautifulsunrise

Beautifulsunrise

    Sĩ quan

  • Thành viên
  • 450 Bài viết
Bài 1(B1): Cho những số thực a, b, c, x, y, z thỏa mãn $a\geq b\geq c>0$
và $x\geq y\geq z>0$. CMR:
$\frac{a^2x^2}{(by+cz)(bz+cy)}+\frac{b^2y^2}{(cz+ax)(cx+az)}+\frac{c^2z^2}{(ax+by)(ay+bx)}\geq \frac{3}{4}.$
Bài 2 (B1):CMR: $\sqrt[3]{\frac{a}{b}}+\sqrt[3]{\frac{b}{a}}\leq \sqrt[3]{2(a+b)(\frac{1}{a}+\frac{1}{b})}$ với mọi số thực a, b.
Dấu "=" xảy ra khi nào?

Bài viết đã được chỉnh sửa nội dung bởi binhmetric: 10-06-2012 - 00:18


#56
le_hoang1995

le_hoang1995

    Sĩ quan

  • Thành viên
  • 314 Bài viết

Bài 1(B1): Cho những số thực a, b, c, x, y, z thỏa mãn $a\geq b\geq c>0$
và $x\geq y\geq z>0$. CMR:
$\frac{a^2x^2}{(by+cz)(bz+cy)}+\frac{b^2y^2}{(cz+ax)(cx+az)}+\frac{c^2z^2}{(ax+by)(ay+bx)}\geq \frac{3}{4}.$

Bài này dùng nhiều quá. Theo thứ tự BĐT AM-GM, Trê-bư-sép, Cauchy-Schwarz rồi đến Nessbit, ta có
$$\sum \frac{a^2x^2}{(by+cz)(bz+cy)}\geq \sum \frac{a^2x^2}{\frac{(b+c)^2.(y+z)^2}{4}}=\sum \frac{4a^2x^2}{(b+c)^2(y+z)^2}$$
$$\geq 4.\frac{1}{3}.\left [ \sum \left ( \frac{a}{b+c} \right )^2 \right ]\left [ \sum \left ( \frac{x}{y+z} \right )^2 \right ]$$
$$\geq \frac{4}{3}.\left [ \frac{(\sum \frac{a}{b+c})^2}{3} \right ].\left [ \frac{(\sum \frac{x}{y+z})^2}{3} \right ]\geq \frac{4}{3}.\frac{\frac{9}{4}}{3}.\frac{\frac{9}{4}}{3}=\frac{3}{4}$$

Bài viết đã được chỉnh sửa nội dung bởi le_hoang1995: 12-06-2012 - 15:14


#57
Maththinkvn

Maththinkvn

    Lính mới

  • Thành viên
  • 1 Bài viết
Thêm bài sử dụng schwars

Cho http://latex.codecog...tex?x,y,z\geq 3 và x +y + z = 3. Tìm Min

http://latex.codecog...tex?x,y,z\geq 3

Bài viết đã được chỉnh sửa nội dung bởi Maththinkvn: 19-06-2012 - 17:25


#58
Luffy9a2

Luffy9a2

    Lính mới

  • Thành viên
  • 1 Bài viết



2)CM bất đẳng thức sau với $x$ là số thực không âm:
$\dfrac{2 \sqrt{2}}{\sqrt{x+1}}+ \sqrt{x} \leq \sqrt{x+9}$



em xí bài 2 nhé :blink:
Bình phương vế trái, ta có
$(\frac{2\sqrt{2}}{\sqrt{x+1}}+\sqrt{x})^{2}$
$\leq (\frac{8}{x+1}+1)(1+x)=(\frac{x+9}{x+1})(x+1)= x+9$

#59
no matter what

no matter what

    Why not me

  • Thành viên
  • 397 Bài viết
em chém bài 2 ,hj bài này rất hay, $\sqrt[3]{\frac{a}{b}}+\sqrt[3]{\frac{b}{}a}\leq \sqrt[3]{2(a+b)(\frac{1}{a}+\frac{1}{b}}$ theo em biết thì cách giải của tác giả bài này là lập phương lên ,cách đó dài,em trình bày cách này nhé :chia vế trái cho vế phải,BĐT tương đương $\sqrt[3]{\frac{\frac{a}{b}}{2(a+b)(\frac{1}{a}+\frac{1}{b}}}+\sqrt[3]{\frac{\frac{b}{a}}{2(a+b(\frac{1}{a}+\frac{1}{b})}}\leq 1$,chú ý rằng $2(a+b)(\frac{1}{a}+\frac{1}{b})=\frac{2(a+b)^2}{ab}$ nên $\sqrt[3]{\frac{\frac{a}{b}}{2(a+b)(\frac{1}{a}+\frac{1}{b})}}=\sqrt[3]{\frac{a^2}{2(a+b)^2}}=\sqrt[3]{\frac{a*a*1}{2(a+b)(a+b)}}\leq \frac{1}{3}\left ( \frac{a}{a+b} +\frac{a}{a+b}+\frac{1}{2}\right )$(theo AM-GM).tương tự rồi cộng vế suy ra dpcm.Em không biết BDT có dung voi moi a=b=c khong nhưng voi a=b=c=1 thì BDT đúng

#60
Beautifulsunrise

Beautifulsunrise

    Sĩ quan

  • Thành viên
  • 450 Bài viết
Cho ${x_1}, {x_2}, ..., {x_n}\,\,(n \ge 2)$ là các số dương thỏa mãn:
${x_1} + {x_2} + ... + {x_n} \le k,\,\,(k\, \in {R^*}),\,b \ge 0,\,\,b{n^2} \ge a{k^2}$.
CMR: $a({x_1} + {x_2} + ... + {x_n}) + b\left( {\frac{1}{{{x_1}}} + \frac{1}{{{x_2}}} + ... + \frac{1}{{{x_n}}}} \right) \ge \frac{{b{n^2} + a{k^2}}}

{k}.$




2 người đang xem chủ đề

0 thành viên, 2 khách, 0 thành viên ẩn danh